LSAT and Law School Admissions Forum

Get expert LSAT preparation and law school admissions advice from PowerScore Test Preparation.

User avatar
 Dave Killoran
PowerScore Staff
  • PowerScore Staff
  • Posts: 5853
  • Joined: Mar 25, 2011
|
#87782
Complete Question Explanation
(The complete setup for this game can be found here: lsat/viewtopic.php?f=157&t=1677)

The correct answer choice is (D)

Answer choice (A): As discussed during the analysis of the grants-to-quarters distribution, three grants in a quarter is possible. To meet the condition in this answer, that quarter would have to be the second quarter, and since that could occur, this answer is incorrect.

Answer choice (B): This answer choice could occur under the 2-2-1-1 grants-to-quarters distribution.

Answer choice (C): Again, this answer choice could occur under the 2-2-1-1 grants-to-quarters distribution.

Answer choice (D): This answer choice cannot occur because the two M grants would be forced into the second and third quarters, a violation of the third rule. Thus, this is the correct answer choice.

Answer choice (E): This answer choice could occur under the following hypothetical:

G4-Q22-d1.png
You do not have the required permissions to view the files attached to this post.

Get the most out of your LSAT Prep Plus subscription.

Analyze and track your performance with our Testing and Analytics Package.